Questions tagged [additive-combinatorics]

Questions on the subject additive combinatorics, also known as arithmetic combinatorics, such as questions on: additive bases, sum sets, inverse sum set theorems, sets with small doubling, Sidon sets, Szemerédi's theorem and its ramifications, Gowers uniformity norms, etc. Often combined with the top-level tags nt.number-theory or co.combinatorics. Some additional tags are available for further specialization, including the tags sumsets and sidon-sets.

Filter by
Sorted by
Tagged with
5 votes
1 answer
482 views

How large can a non-sumset be?

The theory of sumsets $A+B$ where $A$ and $B$ are finite subsets of an additive group $Z$ is extensively studied in additive combinatorics: finding long arithmetic progressions inside them, finding ...
Thomas Bloom's user avatar
  • 6,608
5 votes
1 answer
452 views

Questions on 'Improved bounds for the sunflower lemma'

I have been reading 'Improved bounds for the sunflower lemma' by Alweiss, Lovett, Wu and Zhang (Ann. of Math., Vol. 194(3), 2021), and have some gaps in my understanding of the paper. They are as ...
user avatar
5 votes
2 answers
253 views

The number of co-circular four tuples

Let $A,B ⊂ \mathbb{R}$ such that $|A| = |B| = n$. What is the best-known upper bound on the number of four-tuples in $A \times B$ where the four points are co-circular, they lie on the same circle?
Kim's user avatar
  • 389
5 votes
1 answer
806 views

Density of intersection with shifted sets

Given a subset $S$ of the positive integers $\mathbf{N}$, let $\mathrm{d}^\star(S)$ be its upper asymptotic density, that is, $$ \mathrm{d}^\star(S)=\limsup_{n\to \infty}\frac{|S \cap [1,n]|}{n}. $$ ...
Paolo Leonetti's user avatar
5 votes
1 answer
273 views

Element with unique representation in A+B

Let $A, B \subseteq \mathbb{Z}$ be finite subsets of the integers. Then there exists an element in $A+B$ with a unique representation as a sum of an element in $A$ and an element in $B$, namely $\max(...
user68529's user avatar
5 votes
1 answer
316 views

Primitive recursive bounds for multidimensional polynomial vdW / HJ

In Shelah's paper 679, he proves primitive recursive bounds for the polynomial Hales-Jewett theorem and thus for the polynomial van der Waerden theorem. How about for the multidimensional polynomial ...
Ryan Alweiss's user avatar
5 votes
2 answers
242 views

Progressions in sumset or complement

Fix $\epsilon>0$. For all large $N$, does there exist $A\subset [N]:=\{1,\dots,N\}$ such that both $A+A$ and $A^c:=[N]\setminus A$ lack arithmetic progressions of length $N^\epsilon$? I am aware ...
Zach Hunter's user avatar
  • 3,413
5 votes
3 answers
682 views

How to prove the combinatorial identity $\sum_{k=\ell}^{n}\binom{2n-k-1}{n-1}k2^k=2^\ell n\binom{2n-\ell}{n}$ for $n\ge\ell\ge0$?

With the aid of the simple identity \begin{equation*} \sum_{k=0}^{n}\binom{n+k}{k}\frac{1}{2^{k}}=2^n \end{equation*} in Item (1.79) on page 35 of the monograph R. Sprugnoli, Riordan Array Proofs of ...
qifeng618's user avatar
  • 838
5 votes
1 answer
547 views

Additive energy of random sets

Given two random sets $A,B$ in a finite field (say $x\in A$ independently and with probability $1/2$), what is known about the additive energy $E(A,B)=|\{(a,a',b,b')\in A\times A\times B\times B: a+b=...
Joel Moreira's user avatar
  • 1,701
5 votes
1 answer
354 views

Is the generalized Erdős–Heilbronn problem true for finite cyclic groups?

The generalized Erdős–Heilbronn (GEH) theorem, which is proved by da Silva and Hamidoune in 1994, states that: Theorem. If p is a prime and $X$ is a subset of $\mathbb{Z}_p$, then $|\hat{k}X| \geq \...
Hsien-Chih Chang 張顯之's user avatar
5 votes
1 answer
146 views

Beating trivial bound for $k$-AP-free sets in characteristic $k$

Given integers $k,n\ge 1$, I shall write $\Bbb{Z}_k^n := (\Bbb{Z}/k\Bbb{Z})^n$. Fix $k\ge 3$. Let $r_k(\Bbb{Z}_k^n)$ denote the cardinality of the largest $A\subset \Bbb{Z}_k^n$, such that $A$ does ...
Zach Hunter's user avatar
  • 3,413
5 votes
1 answer
344 views

Is every integer $\ge 312$ the sum of two integers with triangular divisors?

We say that a natural number $n$ has triangular divisors if it has at least one triplet of divisors $n = d_1d_2d_3$, $1 \le d_1 \le d_2 \le d_3$, such that $d_1,d_2$ and $d_3$ form the sides of a ...
Nilotpal Kanti Sinha's user avatar
5 votes
1 answer
176 views

Computational version of inverse sumset question

Let $p$ be prime and $\mathbb{F}_p$ the finite field with $p$ elements. Suppose we have a set $B\subseteq \mathbb{F}_p$ satisfying $|B|<p^{\alpha}$ for some $0<\alpha<1$ and there exists $A\...
user avatar
5 votes
1 answer
160 views

Eccentricity in the number of representations for sets too large to be Sidon sets

Let $A=\{a_1<a_2<a_3<\dots< a_k\}\subset\{1,2,\dots,N\}$ be a set of integers. Let $r_A(n)=\#\{(a_i,a_j):a_i+a_j=n\}$ be the number of representations of $n$ as a sum of two elements from $...
Joseph Vandehey's user avatar
5 votes
1 answer
288 views

Additive basis of order 2

Can we find $\alpha>1$ such that $u=(\lfloor n^\alpha\rfloor)_{n\geqslant0}$ is an additive basis of order $2$ (i.e. $\forall x\in\mathbb{N}, \exists(n,m)\in\mathbb{N}^2, x=u_n+u_m$) ? Remark : ...
uvdose's user avatar
  • 593
5 votes
1 answer
227 views

Unicity of additive, $(-1)$-homogeneous, and shift invariant probability measures on $\mathbf N^+$

Let $\mathcal D$ be the set of all (finitely) additive probability measures $\mu^\ast: \mathcal P(\mathbf N^+) \to [0,\infty[$ such that $\mu^\ast(k \cdot X + h) = \frac{1}{k} \mu^\ast(X)$ for all $X \...
Salvo Tringali's user avatar
5 votes
1 answer
632 views

Cauchy-Davenport strengthening?

Is the following statement, refining classical Cauchy-Davenport Theorem (that states that for sets $A$, $B$ of residues modulo prime $p$, $|A+B|\geq |A|+|B|-1$ provided that RHS does not exceed $p$) ...
Fedor Petrov's user avatar
5 votes
1 answer
269 views

Expected number of coin flips before you see a $k$-term arithmetic progression of heads

Let $\{X_i\}_{i \in \mathbb Z_+} $ be independent fair coin flips. Write $S := \{i \in \mathbb Z_+\, | \, X_i \text{ is heads}\}$, and define, for an integer $k \geq 3$, $$Y := \inf \{n \in \mathbb N \...
Nate River's user avatar
  • 4,832
5 votes
1 answer
274 views

How to generate $n$ FP32 rationals s.t. no two distinct k-el. subsets have same sum?

First some Background: I have lots and lots of integer matrices, whose rows are $k$-combinations (without repetitions and sorted) of numbers from the set $S:=\{1,...,n\}$ and needed to be compared ...
M.G.'s user avatar
  • 6,720
5 votes
2 answers
499 views

Anticoncentration of the convolution of two characteristic functions

Edit: This is a question related to my other post, stated in a much more concrete way I think. I am interested in anything (ideas, references) related to the following problem: Suppose that $A \...
Maciej Skorski's user avatar
5 votes
1 answer
931 views

Minimal polynomial of sums of roots of unity with constant term $\pm1$

Let prime $p$ and given $\zeta_p = e^{2\pi i/p}$. It is well-known that the minimal polynomial of $x = \zeta_p + \zeta_p^{p-1}$ has a constant term either $\pm 1$ and, for certain $p$, the sum of ...
Tito Piezas III's user avatar
5 votes
1 answer
119 views

Maximum size of a critical set that sums to $n$

Say that a set $S \subset \mathbb Z^+$ can express $n$ if there is some way to add elements of $S$ (possibly more than once) to equal $n$. Call $S$ critical if moreover no proper subset of $S$ can ...
wlad's user avatar
  • 4,843
5 votes
1 answer
357 views

Sum-product estimate in finite fields

There is a paper by Bourgain, Katz and Tao Bourgain, Jean; Katz, N.; Tao, Terence C., A sum-product estimate in finite fields, and applications, Geom. Funct. Anal. 14, No. 1, 27-57 (2004). ZBL1145....
Ken Berner's user avatar
5 votes
3 answers
399 views

Request for reference for some proofs about Gowers' norm

For any map $f : \mathbb{F}_2^n \rightarrow \mathbb{C}$ we define its $d^{th}-$Gowers' Norm (for $1 \leq d \leq n$) as, $\|f\|_{U^d(\mathbb{F}_2^n)}^{2^d} = \mathbb{E}_{L : \mathbb{F}_2^d \rightarrow \...
gradstudent's user avatar
  • 2,146
5 votes
1 answer
469 views

Additivity of upper densities with respect to arithmetic progressions of integers

Let $\mathsf{d}^\star$ be the asymptotic upper density, defined on the power set of positive integers $\mathbf{N}^+$, so that $$ \mathsf{d}^\star\colon \mathcal{P}(\mathbf{N}^+) \to\mathbf{R}\colon X\...
Paolo Leonetti's user avatar
5 votes
1 answer
780 views

Balog-Szemeredi-Gowers with dilates of sets

All sets are assumed to be finite subsets of the integers. The additive energy of two sets $E(A,B)$ is defined as the number of solutions to $a+b=a'+b'$ with $a,a'\in A$ and $b,b'\in B$. The well-...
Thomas Bloom's user avatar
  • 6,608
5 votes
1 answer
381 views

Thin subbases for the primes?

Hi all, My question concerns a general problem concern the Erdos-Turan conjecture on additive bases; that of finding thin subbases in a given basis. For a given $A \subset \mathbb{N}$, define $r_{A,h}...
Stanley Yao Xiao's user avatar
5 votes
1 answer
1k views

A question on the singular series and singular integral in Hardy-Littlewood Circle Method

Given a set $A$ of positive integers, set $r_{A,h}(N)$ to be the number of $h$-tuples $(a_1, \cdots, a_h)$ such that $N = a_1 + \cdots + a_h$. Set $f(z) = \sum_{a \in A} z^a$. Then by Cauchy's Theorem ...
Stanley Yao Xiao's user avatar
5 votes
1 answer
265 views

Is there a short proof for the permutation invariance of this combinatorial map?

Consider a positive integer $n$ and integers $(c_i)_{1\le i \le 4}$, with $1 \le c_i \le n$. Conside the map: $$f_n: (c_1,c_2,c_3,c_4) \mapsto \delta_{c_1,c_2}\delta_{c_3,c_4} - \# \{ |2n+1-2|x||, \ x ...
Sebastien Palcoux's user avatar
5 votes
1 answer
372 views

Disjoint union of affine subspaces contains a larger affine subspace

I'd like to say that a large structured subset of the $n$-dimensional Boolean cube $\{0,1\}^n$ contains a non-trivial affine subspace. To be more specific, I want to prove/disprove that for some ...
Alex Golovnev's user avatar
5 votes
1 answer
254 views

Divisibility labeling on a boolean lattice and nonzero Euler totient

Let $B_n$ be the subset lattice of $\{1,2, \dots , n \}$, also called the boolean lattice of rank $n$. A labeling $f: B_n \to \mathbb{N}_{\ge 1}$ is called acceptable if $\forall a,b \in B_n$: ...
Sebastien Palcoux's user avatar
5 votes
2 answers
379 views

Ordered lattice point enumeration

I initially asked this question over at StackOverflow as it has algorithmic flavor to it, but I haven't been getting much traction so I thought I would probe the mathematics community. Setup: Let $e_{...
Paul's user avatar
  • 547
5 votes
0 answers
189 views

Is it true that the $\mathbb{F}_p$-rank of a linear combination of matrices is usually not smaller than its $\mathbb{Q}$-rank?

Consider fixed $3 \times 3$ integer matrices $A_1,A_2,A_3$ and the $\sim H^3$ linear combination matrices $A(\mathbf{h})=h_1A_1+h_2A_2+h_3A_3$ where $h_1,h_2,h_3$ are integers with $\vert h_i\vert \le ...
Christian Bernert's user avatar
5 votes
0 answers
77 views

Maximum size of difference sets with a bounded number of prime divisors

Call a subset $S\subset \mathbb{Z}$ $r$-smooth if the difference set $S-S$ contains numbers whose prime divisors lie in a set $P$ of distinct primes with $|P|=r$. Let $f(r)$ be the maximum size of any ...
Ivan Meir's user avatar
  • 4,792
5 votes
0 answers
191 views

Showing that Fourier pseudorandomness is insufficient for $k=4$ case (four arithmetic progressions)

I wish to show that the Fourier pseudorandomness is insufficient to count the number of 4-term arithmetic progression. Let $A \subset \mathbb{Z}/N\mathbb{Z}$ be a subset of a cyclic group $\mathbb{Z}/...
Killua Zoldyck's user avatar
5 votes
0 answers
197 views

Ramsey Numbers for Integers

Erdos defined $f(n)$ to be the minimum $r$ such that there is an $r$-coloring of the positive integers less than $n$, wherein $n$ cannot be written as the sum of distinct monochromatic integers. ...
Yoozer2021's user avatar
5 votes
0 answers
114 views

Low-rank approximation over finite fields

Consider the finite field $\mathbb{F}_p$ with prime $p$. Let $I=\{ 0,1,2,...,|I|-1 \}\subset \mathbb{F}_p$ be an "interval". What can be the largest size $|I|$, such that there exists a $2\times 2$ ...
fil's user avatar
  • 51
5 votes
0 answers
173 views

Large finite subsets of Euclidean space with no isosceles (or approximately isosceles) triangles

Here's a question in combinatorial geometry which feels very much like other questions I'm familiar with but which I can't see how to get a hold of. I'll actually propose two different questions on ...
JSE's user avatar
  • 19.1k
5 votes
0 answers
336 views

Lower bound for some sums of roots of unity

Let $n$ be a positive integer (assume $n$ is prime for simplicity), and let $x_k = \pm1$, for $k = 0,1,2,..., n-1$. Let $\rho$ be an $n-$th primitive root of unity, I am interested in a lower bound ...
shurtados's user avatar
  • 1,020
5 votes
0 answers
195 views

Is every integer $n>1$ the sum of two triangular numbers and two powers of $5$?

Recall that the triangular numbers are those integers $$T_n=n(n+1)/2\ \ \ (n=0,1,2,\ldots).$$ In 1796 Gauss proved that each $n\in\mathbb N=\{0,1,2,\ldots\}$ is the sum of three triangular numbers, ...
Zhi-Wei Sun's user avatar
  • 14.5k
5 votes
0 answers
113 views

$m$-thick sets with small $n$-fold sumsets in finite cyclic groups

Problem. Is it true that for every positive integers $n,m$ there exists a subset $A_{n,m}$ of a finite cyclic group $G$ having the following two properties: $(\Sigma_n)$ the $n$-fold sum $A_{n,m}^{+...
Taras Banakh's user avatar
  • 40.9k
5 votes
0 answers
833 views

Increasing sequences in polynomial progressions modulo p

In a random permutation on $n$ elements one expects the largest increasing and decreasing sequences to have size $(2+o(1))\sqrt{n}$. Is it known if this same property holds in sequences given by ...
Ryan Alweiss's user avatar
5 votes
0 answers
153 views

Negative values of cosine sums

Consider real numbers $x_1, \dots, x_M$ such that $$\sum_{i=1}^{M} \frac{\cos(x_i t^2)}{e^{(x_it)^2}} \le -\frac{1}{2}, $$ for all $L< t <L^A,$ where L is a large number. What lower bound ...
Farzad Aryan's user avatar
5 votes
0 answers
79 views

Some questions about the Lévy monoid of certain densities

Let $\bf H$ be a set, $f: \mathcal P({\bf H}) \rightharpoonup \bf R$ a partial function, and $\mathcal{D}$ the domain of $f$. Next, denote by $\mathcal M(f)$ the set of all (total) functions $\theta: ...
Salvo Tringali's user avatar
5 votes
0 answers
239 views

The sum of all the elements of every non empty subset of $A$ is not a multiple of $n$

Let $N=\{1,2,\ldots ,n\},n>1$. We wish to construct a set $A\subseteq N$ with the property: The sum of all the elements of every non empty subset of $A$ is not a multiple of $n$. Question: ...
Konstantinos Gaitanas's user avatar
5 votes
0 answers
247 views

"Pseudo-random" subsets of additive bases

We say that a subset $B \subset \mathbb{N}$ is an (asymptotic) additive basis of order $k$ if the set $kB = B + \cdots + B = \mathbb{N} \setminus C$, where $C$ is a finite set of positive integers. ...
Stanley Yao Xiao's user avatar
5 votes
0 answers
434 views

A question about Erdős thin bases

Let $B \subset \mathbb{N}$ be an additive basis of order $h$. Define $r_{B,h}(n)$ to be the number of ways $n$ can be written as a sum of $h$ elements of $B$. In particular, $B$ is a basis of order $h$...
Stanley Yao Xiao's user avatar
4 votes
1 answer
1k views

a conjecture in sum-free sets

Let $ A $ be a set of non-zero integers. Then $A$ contains a sum-free subset $B$ of size $ |B|> \frac{|A|}{3} $ (a result of Erdős). It is conjectured that RHS can be improved to $\frac{|A|}{3} +...
Koushik's user avatar
  • 2,076
4 votes
2 answers
504 views

Average involving the Euler phi function

Does $$\frac{1}{N^2}\sum _{d=1}^N \log d \sum _{n=1}^{N/d} \frac{\phi(n)}{\log (dn)},$$ converges or not when $N$ goes to infinity?
wongpin101's user avatar
4 votes
2 answers
529 views

Density version of the Erdos-Graham conjecture

In 2003 E. S. Croot [Ann. of Math. 157(2)(2003), 545-556] proved the Erdos-Graham Conjecture which states that if $\{2,3,\ldots\}$ is partitioned into finitely many subsets then one of the subsets ...
Zhi-Wei Sun's user avatar
  • 14.5k

1
4 5
6
7 8
14